You are on page 1of 39

I.

Property Outline
What is Property?
A. The Right to Exclude
1. Jacque v. Steenberg Homes (p. 4) D intentionally treks trailer home purchased by
Ps neighbor across Ps property against Ps wishes.
a. P has right to refuse entry onto his property to D
b. loss of individuals right to exclude may be punished by punitive damages
despite the lack of measurable harm
i. punitive damages appropriate for intentional trespass b/c they constitute
deterrence and allow enforcement of trespass law
c. Holding: P awarded punitive damages for intentional trespass by D
2. State v. Shack (p. 12) - social worker and attorney trespass on Tedescos property to
assist migrant workers
a. Limitations on Right to Exclude for emergency and necessity
i. conflict between Ds property rights and rights of migrant workers
b. title to real property does not include dominion over destiny of persons the owner
permits to come upon premises
c. Holding: Ds right to exclude does not include right to bar access to
governmental services available to migrant workers, and hence there was no
trespass by P
3. Intel v. Hamidi (p. 20) disgruntled former employee used Ps email system to send
emails criticizing P
a. Trespass to chattels: interference w/ anothers possession or use of personal
property
b. damages for trespass to chattels only appropriate if there is tangible harm to
property
i. no damage to Intels email system b/c Ds emails constituted a small portion
of the total emails on system
ii. Intel claimed there was tangible damage in the loss of productivity from
employees having to delete Ds messages
(i) Should Intel have to wait until it is harmed to seek injunctive relief?
(ii) Should Intel be able to control the use of its email system?
(iii) This court said no to both
c. Holding: Ds use of email system did not constitute trespass of chattels b/c there
was no harm to Ps property (email system) or to Ps use of its property
i. D won the right to use email system, but not to overburden it
B. Different Types of Property
1. Pierson v. Post (p. 34) Pierson kills and carries off fox on public beach, knowing that
Post was already in pursuit of fox
a. first in time vs. second in time person
i. first in time to capture more reasonable, b/c its impossible to determine if
Post would have captured the fox
b. fox as vermin
i. courts want to encourage hunting and killing
c. must have actual possession of object to constitute ownership
i. pursuit alone does not equal ownership
d. constructive possession: person does not actually have possession, but law
constructs possession of the object
e. Holding: Fox was property of Pierson, who intercepted and killed it. Piersons
act in capturing fox constituted no wrong towards Post, and Post not entitled to
damages.

2. Keeble v. Hickeringill (p. 40) P owns pond w/ decoys to lure ducks as his
enterprise. D intentionally fires shots to scare ducks from Ps duck pond;
a. D had malicious intent
b. D unreasonably interfering w/ Ps enterprise
i. If D had interfered w/ Ps business by setting up his own competing business,
no action would lie
c. Holding: D liable for trespass and P awarded damages for disturbance caused by
Ds trespass
3. Popov v. Hayashi (p. 46) Dispute over ownership of Barry Bonds record homerun
ball. P first to stop balls motion, but after crowd brawl, D ultimately ended up with
ball
a. Different view as to what constitutes possession
i. first person to stop ball;
ii. first person to come into possession of ball; or
iii. first person to obtain complete control of ball
b. P took significant but incomplete steps to achieve possession and his effort was
interrupted by the unlawful acts of others,
i. So P had legal cognizable pre-possessory interest in property
c. Holding: Mob interfered w/ Ps possible possession, so court split ownership
between P and D
i. P and D ordered to sell ball at auction and split the proceeds
4. Anderson v. Beech Aircraft Corp. (p. 54) D injected gas into reservoir under Ps
and Ds properties. P landowner and P lessee under oil and gas lease brought action
to quiet title to gas P lessee produced from reservoir.
a. Ps lessee has right to retrieve non-native gas from their property
i. Gas previously injected into common reservoir by D, another landowner
ii. D had no license or lease covering land from which non-native gas is being
produced
iii. Lessee has possessory rights to gas from P through lease
b. D lost its ownership right after injecting gas into reservoir
c. natural gas is not property until it is removed from the ground
d. Holding: Ps entitled to produce non-native gas from reservoir beneath their
property.
II. Chapter 2: Acquisition of Property by Conquest, Adverse Possession, and Prescription
A. Conquest
B. Johnson and Grahams Lessee v. McIntosh (Page 116) Indians had possession of land
and transferred to Johnson & Graham in 1775 and want to quiet title. Govt had title to
land and transferred to McIntosh in 1819
1. Discovery vs. Conquest
a. Discovery gave Europeans right to extinguish Indian title of occupancy by
discovery, but until then Indians still had title
b. Conquest acquired by force; U.S. claimed title to land w/in its territories
i. deemed Indians incapable of transferring title of property to others
2. Holding: Indians transfer of title to McIntosh invalid b/c Indians didnt have right to
transfer title at that point in time; land belonging to U.S. govt
C. Adverse Possession and Prescription of Interests in Land
D. Ewing v. Burnet (Page 128):
J. Symmes transferred land 6/11/1798 Foreman 1798 Williams 1824 Ewing;
J. Symmes transferred same property to Burnet on 5/21/1803.
1. Burnet dug sand and gravel from lot and leased property to others to do the same, but
did not occupy the lot

a. Claimed he owned lot and paid taxes on lot from 1810 1834
2. Williams knew of Burnets possession of property and made no effort to enter land or
demand possession
3. Burnet had color of title even if he didnt have the actual title (was 2 nd in time
grantee)
a. Court more concerned w/ intent to occupy/use land than w/ actual titles
b. Doesnt matter that Ewing didnt acquire title until after Burnet had been
occupying land
4. Holding: Burnet had occupation of land for statutory period (21 years), SOL ran out
and Burnet acquired title by AP
E. Different standards for determining claim of right:
1. Why do courts care if occupant of property is aware that he owns property or not?
a. aggressive trespass standard (bad-faith): rewards those who have connection to
land, invested labor to develop land; want aggressive trespasser rather than
someone who doesnt know what hes doing w/ land
b. objective: we dont care what youre thinking, objective acts speak for
themselves; administrative ease
c. good faith: if you have color of title (written document upon which claim is
founded), good faith is presumed
F. Adverse Possession
1. Way to acquire title to property you dont actually own by possession
2. Purposes
a. Protects title b/c actual ownership can be difficult to prove
b. rewards those who use land productively
3. Four Requirements for Adverse Possession:
a. Actual entry of land and acquiring exclusive possession not sharing possession
w/ general public
b. Open and notorious reasonable notice that adverse possessor is claiming
dominion over property
i. Gives owner knowledge of AP
c. Adverse/Hostile to owner - w/o owners consent and not subordinate to owner
i. Objective test: APs acts must look like claims of ownership but AP is not
actually claiming title
ii. Subjective test: AP must have good faith belief that he has title
iii. Color of title: claim founded on written instrument not required
(i) Period required to establish title by AP shorter
(ii) Adverse possessor regarded as having constructive possession of
property described in deed
iv. Boundary disputes may lead to AP depending on jurisdictions requirement
regarding whether possession of land on boundary is intentional, adverse or
mistaken
d. Continuous, uninterrupted possession AP must make same use of property an
ordinary owner would
i. Seasonal use constitutes continuous use
4. Statutory Period, Tacking and Problems in Applying the SOL (p. 147-8)
a. When AP comes onto property owned by someone else, AP takes a number of
risks in addition to whether she is satisfying the above requirements
b. If owner has disability, statutory period for AP may be extended to a add on
certain number of years after disability has lifted
i. Owner has benefit of whichever time period is long
c. Owner cant tack on disabilities after AP has taken possession of property. (Ex.

if O dies and leaves property to heir that is a minor, but AP was already on
property, then disability doesnt matter)
d. Adverse Possessor can tack on period of AP to previous predecessors in interests
period of AP
5. Future interests not affected by AP
a. Remainders interests not affected by AP
G. Nome 2000 v. Fagerstrom (Page 131) Ds build summer home on property owned by
D.
1. Issues:
a. Dispute between land claimed to be used by D and actual portion of Ps property
occupied by D; D only used small portion of Ps property
b. D had Actual possession: not just placing belongings all over property (junkyard)
to constitute notice to owner of record
c. Open and notorious: D established physical occupation by presence of camper,
shelter, picnic area, non-native trees
d. Exclusivity: D let other people use trails on property, but kept people from
burning Ds firewood; were acting as hospitable landowners
e. Trails already existed before Ds came on property, possibly evidence that Ds
didnt adversely possess portion of property w/ trails
f. Continuous possession: D built cabin in 78, relevant period of continuous
possession is 1978 1987 for SOL
i. seasonal occupation by D used land only in summer
ii. important what other people (not Nome 2000) who encountered property
would think - would they consider D acting as landowner?
iii. Only occupying property for part of year can still lead to AP
2. Holding: Ds evinced purpose to exercise dominion and control over property. Ds
acquired title to land they actually occupied by AP, but not to other portion of
property not occupied by D.
H. Lawrence v. Town of Concord (Page 138) Harriet, testator, died and left property in her
will to daughter (life estate), then adoptive daughter (life estate), then to children of
adoptive daughter (contingent remainder), then to town when adoptive daughter dies (if
adoptive daughter doesnt have children or children dont survive her)
1. Testators adoptive daughter died and P, Harriets husband, takes control of property
2. Town not aware that they own property; No requirement that owner given notice of
adverse use of his property
a. No requirement that owner is aware of his ownership
3. Husband lives on property, rents property for some time period, and pays taxes on
property
a. P not owner of property, property couldnt have been left to him as an heir to his
wife b/c land was life estate and ownership ended when wife died
4. City couldve reasonably found out that they owned property by searched public
records b/c conveyance was in will
5. Holding: Husband occupied land w/o permission of owner, continuously for statutory
period, exclusively, openly, notoriously and adversely to true owner, and thus
acquired title by AP.
I. Cant adversely possess against the govt
1. difficult and costly for govt to monitor all its properties
J. Kiowa Creek Land & Cattle Co. v. Nazarian (Page 149): State of Nebraska NE Board
of Educational Lands and Funds (NBEL&F) Leased to Nazarians, then sold to
Nazarians
1. AP (Kiowa) comes on property when Nazarians are leasing land from govt and

remains for statutory period


2. AP claims it has acquired prescriptive easement b/c it has possessed against same
people for entire time
a. Prescription (acquiring easement) vs. AP (fee simple absolute)
3. Cant adversely possess against government, so why is this case even an issue?
a. Nazarians didnt act in good faith as owners when they were lessees to prevent
use of property by P
b. Nazarians shouldve stopped Ps expectations from accumulating
4. Holding: No AP, b/c land was owned by govt when AP entered property, SOL
doesnt run against govt, and D hadnt possessed land for statutory period notincluding time when land was owned by govt.
K. Dieterich International Truck Sales, Inc. v. J.S. & J. Services, Inc. (Page 152) P seeks
to quiet title to easement across Ds truck stop. D is lessee, not owner, of property.
1. Issue: Can a landlord lose his f.s.a. ownership in his property through a tenants
failure to stop an adverse possessor?
a. General Rule: if AP enters after lessee takes possession, AP is possessing against
lessee, not owner
i. AP gets property interest in lease, not ownership
b. Does owner have right to prevent adverse possession of lessees interest?
i. Waste law enables owner to protect property during lease
ii. Statute provides that person having future interest (e.g. landlord) may
maintain actions to protect injury done to inheritance, even though he doesnt
have current interest in property
iii. Perhaps burden should be on lessee, b/c lessee has actual possession of
property and greater ability to stop AP
2. Holding: P achieved adverse possession against tenant, but not against owner
III. Property Rights in Creative Works and Human Tissue
A. Property Rights in Personas and the Right of Publicity
1. Right of publicity: protects person's interest in commercial exploitation of their
name, likeness, and identity
a. Doesnt apply to use of name in news reporting, unauthorized biography,
entertainment parody/satire
2. Wendt v. Host International Inc. (p.192)-Actors appealing summary judgment for
D's use of likeness without permission for robotic images in Cheers airport bars
a. Court held that issue of whether figures are similar enough to Ps to
constitutes "likenesses" issue for jury to decide
b. Should the law be designed just so actors get money?
c. What does it cost us? Limits ability to make spin-offs/sequels
d. Rights of copyright holder vs. actors rights of publicity
i. Perhaps D would have been better off using exact image of actors, would
be using the Cheers characters (owned by Cheers copyright holder), and
not accused of trying to invoke actors likenesses
3. Zacchini v. Scripps-Howard Broadcasting Co. (p. 201)- Human cannonball act
brought an action against a TV broadcasting co. to recover damages for playing
entire performance on a TV news program, against Zs wishes
a. 1st and 14th Amendments dont immunize media when they broadcast a
performers entire act w/o consent
b. Broadcasting entire act threatens economic value of performance
i. Protection provides artists incentive to product performances of interest to

public
How far is right to exclude?
i. should other cannonball performers pay a licensing fee to Z how creative
is the process
d. Holding: Z entitled to monetary damages for unlawful appropriation of his
professional property
B. Property Rights in Writings, Recordings, and Product Designs
1. Copyright Act - 70 years of protection for original literary, dramatic, musical,
artistic, intellectual works published and unpublished
a. Protects expression of idea, not idea itself exclusive right to work you
produce
b. right to exploit the work for profit and build derivative works
c. Provides economic incentive for authors to publish books and disseminate
ideas to public encourage creation of original works
d. Ensures works enter public after author's rights have expired
2. Fair Use Doctrine:
a. Balances constitutionality of copyright laws w/ 1st Amendment values
b. Allows use of work for purposes of criticism, parody, comments, news
reporting, teaching, research -- not copyright infringement
c. Considerations:
i. commercial or nonprofit educational uses
ii. nature of copyrighted work
iii. amount of portion used from original work
iv. effect of use on potential market or value of copyrighted work
d. Suntrust Bank v. Houghton Mifflin Company - Owners of copyright in novel
"Gone With the Wind" brought action for TRO and preliminary injunction to
prevent publication and distribution of allegedly infringing book "The Wind
Done Gone"
i. TWDG had element of creativity but needed to evoke images of characters
ii. TWDG could enhance value of GWTW b/c have to read one to understand
the other
iii. Holding: TWDG met fair use defense b/c work transformative enough as
parody
C. Trademark and Tradedress
1. Trademark: controlled by Lanham Act
a. have perpetual right to renew license, exclusive right to use and register
trademark and to prevent others from using it
b. Primary purpose is to protect consumers and avoid consumer confusion
i. Allows companies w/ good products to reap benefit of customer loyalty
c. Protection afforded by registered trademark:
i. Standing to bring cause of action in state & fed. courts
ii. Gives constructive notice to world of ownership
iii. Get notice from customs re: importation of similar goods (knock-offs)
iv. Can prevent dilution of trademark
d. Secondary Meaning: of something used to signify product
e. Doctrine of Functionality: No aspect of trademark should be functional
i. Trademarking functional attributes Impedes development
ii. Must prove that trademark protection, and not patent protection is
warranted
2. Tradedress: appearance of image of company has represented in some packaging
design; not registered
c.

3. Qualitex Co. v. Jacobson Products Co. (p. 249)- Manufacturer of press pads used
in dry cleaning and laundry establishments brought action against competitor
alleging trademark infringement and unfair competition when competitor copies
green-gold color for pressing cloth
a. Issue: Can color be a trademark?
b. If functional, then no; not functional now, so ok
i. Could be problem if all the good colors were already taken (green
pancakes)
c. Holding: (1) no special legal rule prevents color alone from serving as
trademark, and (2) green-gold of manufacturer's dry cleaning press pads could
be registered as trademark
4. Traffix Devices, Inc. v. Marketing Displays, Inc. (p. 254) MDI, maker of
Windmaster sign stands, brought trademark and tradedress infringement action
against Traffic that used WindBuster mark for its traffic sign stands
a. MDIs patent expired, so no claim to functional attributes, but claims spring
design has taken on secondary meaning
i. MDI had utility patent for spring design, signifying it was functional
b. Legal definition of functional: Exclusive use of attribute would put
competitors at non-reputation related disadvantage
c. Lay definition: springs necessary to function of device
d. Holding: Springs functional, no tradedress protection (won on Trademark
claim for similar names)
D. Patent Law
1. Grants inventors right to prevent others from making, using, or selling invention
for limited time period
2. Eligibility: must be novel, usefully, non-obvious in nature
a. Limits: laws of nature, physical phenomena, abstract idea
3. Term: begins on date issued and ends 20 years from when application filed
a. exception: design patents: 14 years from date issued
b. not renewable, making it more available fro public use
c. if modify enough can get patent extended (drug companies do this)
4. Three types
a. Utility: new and useful machine, manufacture process, composition in matter
b. Design: new, original, ornamental design for an item or manufacture where
only appearance and not actual item protected
c. Plant: new asexually reproducing plant
5. Diamond v. Chakrabarty (p. 262) - Issue: Does a living micro-organism
constitutes a "manufacture" or "composition of matter" within the meaning of the
patent statute?
a. respondent's claim nonnatural occurring manufacture/composition of matter
producing new bacterium so should be patentable
b. Holding: Congress intended wide scope for patent protection
i. Courts decision stimulated research in many new areas
E. Property Rights in Human Tissue
1. Conversion: tort that protects against interference with possessory and ownership
interests in personal property
2. Uniform Anatomical Gift Act (UAGA): Allows next of kin right to transfer parts of
bodies in possession to others for med. or research uses
a. Prohibits sales only for transplantation, therapy, or reconditioning
b. Moore v. Regents of the University of California (p. 266) - P not told by
doctor that his cells would be used to develop cell line. P brought action

against physician, university, etc. alleging conversion and breach of


physician's disclosure obligations
i. Holding: patient had cause of action for breach of physician's disclosure
obligations
ii. No cause of action for conversion
(i) Balancing of policy concerns: Small inconvenience to patients vs.
potential huge gain to society
(ii)Would chill research
1. liability on anyone who came into contact w/ cell line?
(iii)
problems better suited for legislative resolution
(iv)
tort of conversion not necessary to protect patients' rights
(v) morality of commodification of body if property, then is it
marketable?
(vi)
Did patient really contribute anything?
iii. Dissent: even given information, no reasonable patient would walk away
when facing terminal illness, not realistic that still has autonomy
(i) patient has right to consent, but not to share in profit
c. Newman v. Sathyavaglswaran (p. 284)- Parents of deceased children brought
action against county coroner's office alleging deprivation of property w/o due
process (14th amendment), based on removal of children's corneas w/o notice
or consent
i. Statute: coroner can remove as long as he has no reason to believe there
will be objection
ii. Inverse condemnation? Government taking property w/o just
compensation?
iii. UAGA interpretation shows property interest rights and of parents in
bodies of children to refuse to allow transfer
iv. Court disregards CA labeling of "quasi property" for next of kin
v. Holding: exclusive right of next of kin to possess bodies of deceased
family members created property interest entitled to due process protection
(i) some kind of consent for due process needed but doesn't define
IV. Chapter 5 - Estates in Land and Future Interests in the United States Today
A. Four types of present possessory estates
1. Fee simple estate - owner has the right to possession into the indefinite future;
potential of enduring forever.
a. To A and his heirs. Modern times - To A is sufficient to create a fee
simple.
2. Lesser estates - fee simple owner gives up possession for a period of time:
a. Fee tail estate - inheritable only by the lineal descendents of the person to
whom it is granted. Limited inheritability. If the lineal decedents die out
then the property is returned to the original grantor or his heirs.
i. Why? To keep property in the family.
ii. To A and the heirs of his body.
b. Life Estate - Expires at the death of the person to whom it is granted.
i. To A for life.
ii. Why? To provide for an elderly parent, spouse, etc.
c. Term of years - Lease. Expires at a specified date. Why? Owner wants to
get an income stream (rent)
B. Language Used to Create Estates: Words of Limitation and Words of Purchase
1. Words of purchase - identify the person to whom the estate is created.
a. To A and her heirs. To A are words of purchase, identifying A as the taker.

2. Words of limitation - words describing the type of estate created.


a. To A and her heirs. and her heirs are words of limitation, indicating a fee
simple.
C. Who Are the Heirs? Inheritance of Land
1. To A and his heirs heirs have no interest in the property. A can sell or give
away the fee simple without his heirs consent.
2. Heirs take the probate estate of a person without a valid will (dies intestate).
3. Heirs include surviving spouse, decedents issue (lineal decedents), parents and
grandparents, etc (ascendants), and other blood relatives (collaterals- siblings,
uncles, aunts, cousins).
4. If a will is left (dies testate), the persons who are devised land are called devisees.
5. What if there are no heirs? Escheat
a. If no will and no heirs, land goes to the state in fee simple.
D. The Defeasible Fee Simple: Controlling Land Use and Behavior by Threat of Forfeiture
Fee simple granted on a condition, potential for infinite duration but not the certainty.
1. Three Types of Defeasible Fees
a. Fee simple determinable - The grantee holds a fee simple until a condition
happens, when the grantees estate terminates and the fee simple
automatically returns to the grantor.
i. Until the condition happens, the grantors interest is called a possibility of
reverter.
ii. Common words - so long as, while, during, until, unless
iii. May be transferred or inherited as long as the stated event has not
happened. But the fee simple remains subject to the limitation no matter
who holds it.
b. Fee simple on condition subsequent - does not automatically terminate but
may be cut short (divested) at the grantors election when the stated condition
happens.
i. grantors interest is called a right of entry or power of termination.
ii. Common words - Provided that, on condition, but if
iii. Does not automatically end. Estate continues in the grantee until the
grantor exercises her power of reentry and terminates the estate. Grantor
has the option of whether or not to exercise this power.
iv. Preferred to a fee simple determinable b/c forfeiture is optional
v. Example - To A, but if liquor is ever sold on the premises, the grantor has
the right to reenter the premises.
c. Fee simple on executory limitation - A fee simple that, on the happening of
the stated event, is automatically divested in favor of a third party.
i. Until the condition happens, the named third partys interest is called an
executory interest.
ii. Common words - until (or unless), then to, but if, then to
iii. Automatically ends like a fee simple determinable.
iv. Example: To School Board, but if within the next 20 years Blackacre is
not used for school purposes, then to A.
d. Station Associates, Inc. v. Dare County (Page 326) - In 1897, Etheridge
conveyed land to the U.S. for use as life-saving stations. In 1989, the U.S.
abandoned the station and in 1992, the U.S. quitclaimed its interest in the
property to Dare County (D). Ps, heirs of the original grantor, Etheridge,
claimed title to the property.
i. Issue: Did the 1897 deed convey a fee simple determinable or a fee simple
absolute?

ii. Deed is devoid of express language of reversion or termination. Nowhere


does the deed indicate that the United States interest in the property would
automatically expire or revert to the grantor upon the discontinued use of
the property as a life-saving station.
iii. Holding: A fee simple absolute
e. Red Hill Outing Club v. Hammond (Page 332) Ds conveyed Red Hill to
the club (P) in a fee simple subject to a condition subsequent that the club
wouldnt fail to provide skiing facilities to town residents for a period of 2
years. The club ceased offering free ski lessons and did not obtain a rope tow
permit for a two season period. Ds filed a notice of reentry and possession.
i. Issue - Should the condition subsequent be strictly construed? (Did they
need to just maintain and make available the premisesas a ski slope,
or did they need to offer free ski lessons, operate a ski tow, etc.?)
ii. General rule to determine the parties intent at the time of the conveyance
in light of the surrounding circumstances, but does not apply to conditions
subsequent.
(i) Conditions subsequent are viewed with disfavor b/c of their potential to
cause forfeiture of land and their adverse effect on the lands
marketability and development.
iii. Holding - Strictly construed.
2. Restraints on Alienation and Unenforceable Conditions
a. Absolute restraints - usually void. E.g. To A in fee simple, but if A transfers
the property without the grantors consent, the grantor shall have the right to
reenter and terminate the estate granted.
b. Partial restraints - often void, but may be held valid if the restraint has a
legitimate purpose and its impact and duration is reasonably limited.
c. Penalties on marriage - void if it penalizes marriage but valid when the
purpose is to give support until marriage.
d. Public policy/social good
3. City of Palm Springs v. Living Desert Reserve (Page 337) - City granted land to
maintain a desert preserve and equestrian center. After less than three years, City
decides that it would rather build a golf course on the land. City tried to buy the
Living Deserts reversionary interest in the land. When the Living Desert declined,
the city decided to take the interest by eminent domain.
a. Issue: Can the Living Desert, the future interest holder, get just
compensation?
b. General rule is that when a condemnor takes property the ownership of which
is split into an estate of fee simple subject to a condition subsequent and a
power of termination, the owner of the future interest is not entitled to any
compensation unless the condition has been breached as of the date of
valuation.
i. Exception - reversionary interest is compensable if the reversion would
have been likely to occur within a reasonable short period of time.
(i) Evidence indicates that violation was imminent.
ii. Also, the grantee of the present interest and the condemnor are one and the
same.
c. Holding: Living Desert entitled to just compensation
4. Differences Among the Defeasible Fees: Adverse Possession, Alienability, and
Duration
a. Adverse possession
i. Fee simple determinable - when the condition occurs the holder of a

possibility of reverter immediately and automatically becomes the owner


of the property in a f.s.a., and the possesor becomes an AP.
ii. Fee simple on executory limitation - when the condition occurs, the holder
of the executory interest immediately and automatically becomes the
owner of the property in a f.s.a. and the possessor becomes an AP.
iii. Fee simple subject to a condition subsequent - title does not change until
the grantor exercises power of termination. Even after the occurrence of a
condition, the holder of the fee simple on condition subsequent remains a
rightful possessor.
(i) Limitations if grantor unduly delays the right of entry.
b. Duration
i. Common law - Possibilities of reverter and powers of termination retained
by the grantor not subject to RAP; Executory interests were subject to the
Rule.
ii. Different depending on the state - some states - require a notice of intent to
preserve
E. Life Estates and Remainders: Providing for the Family
1. Types: (a) For life of grantee; (b) Pur autre vie - measured by someone other than
the owner of the life estate; (c) In a class - To the children of A for their lives,
remainder to B.
2. Defeasible life estates
a. Nelson v. Parker (Page 345) - Russell Nelson created a life estate in himself
and a remainder in Daniel Nelson in a f.s.a. The problem is the deed is
subject to a life estate in Irene Parker.
i. Issue: Does a deed subject to a life estate in a third person (Irene) validly
create that life estate?
ii. Common law rule - a grantor cannot reserve a life estate in real property to
a party who is a stranger to the deed. Need to convey it in a two step
process.
(i) Rule no longer serves a practical purpose, so should look at the
grantors intent which is clear in this case.
iii. Holding - It does - overrules earlier authority.
b. In re Estate of Kinert v. Pennsylvania Department of Revenue (Page 349) Mrs. Kinert died and left a questionable clause in her will concerning the
disposition of her home.
i. Issues: 1) Did Mrs. Kinert intend to devise a life estate or to bequeath a
license to her foster sons? 2) Can the sons interest in the property taxed?
ii. Intention must be construed from the language of the will - Terminable
life estate , devise
(i) Life estate even though there are limitations on sons
iii. Holding: It is a life estate. It can be taxed. \
c. In re Estate of Jackson (Page 353) - Jackson conveyed a life estate in herself
and a remainder in f.s.a. to Miller and Brosnan. When Jackson was still alive
a hail storm caused substantial damage to the house.
i. Issue: Insurance proceeds for hail damage came in after Jacksons death.
Who should receive the insurance proceeds and how should they be used?
(i) Should they go to Jacksons estate or to successors in interest to
property?
ii. As a life tenant, Jackson could do no act to the injury of the inheritance.
She was responsible for the general maintenance of the house and to make
any necessary repairs to the house.

iii. Holding: Money should go to her estate and then from her estate to pay for
the damages to the house.
d. Hausmann v. Hausmann (Page 355) - Esther Buckley deeded her son
George Hausmann a life estate in her property, with a remainder to Charles
Hausmann. George wanted a f.s.a. so upon the advice of his attorney stopped
paying real estate taxes for the property in an attempt to divest Charles of his
interest in the land.
i. Issue: Does not paying taxes equal waste? Is issuing an injunction a
proper remedy?
ii. Holding: Failure to pay real estate taxes on a life estate by the life tenant
may give rise to a cause of action in waste. An injunction would not be an
improper remedy. Award of punitive damages is also allowed.
iii. Reasoning - Waste occurs when someone who lawfully has possession of
real estate destroys it, alters it or neglects it so that the interests of persons
having a subsequent right to possession is prejudiced in some way or there
is a diminution in the value of the land being wasted.
e. Waste - rules governing the life tenants responsibilities for care and
conservation of the property.
i. Kinds of waste:
(i) Affirmative (voluntary) waste - results from intentional acts of the life
tenant that cause substantial damage to the value or character of the
property.
(ii)Permissive (Involuntary) waste - results from failure to make repairs or
to pay the ordinary carrying charges of the property.
(iii)
Ameliorating waste - life tenant increases the value of the
property by making permanent changes in its use substantially altering
structures, or building additional structures on the property.
3. Alienability of Life Estates - Fully alienable unless subject to a valid restraint on
alienation.
a. Life tenant cannot convey more than she has - an estate that ends when she
dies. A life tenant cannot sell the property unless the remaindermen or
reversioners are willing to join in the conveyance.
i. court may order a sale when the property cannot generate sufficient
income to pay the carrying charges or make repairs necessary to maintain
its value to protect the interests of both the life tenant and the future
interest holder.
ii. Problems arise when the future interest holder is has not yet been born or
identified.
F. Remainders, Reversions, and Executory Interests (Future interests)
1. Indefeasibly vested - A future interest is indefeasibly vested when there are no
conditions attached to it and it is owned by an identified person.
a. E.g. O to Irene for life, remainder to Daniel. or O to Irene for life.
2. Vested Subject to a Complete Divestment - A future interest is subject to complete
divestment if, on the occurrence of a condition, the interest will be shifted to
someone else.
a. Remainder - Will happen if there is an express condition subsequent attached
to the remainder.
b. Reversion - Will happen when a condition happens that causes another,
previously contingent, interest to vest.
c. E.g. O to Irene for life, remainder to Daniel if he survives Irene.
d. Contingent - A future interest is contingent when it is subject to a condition

precedent, which is a condition that must happen before the remaindermans


interest vests.
i. A future interest is ALWAYS contingent if the remainderman has not yet
been born or has not been identified.
ii. Remainders are often contingent and executory interests are nearly always
contingent. E.g. O to Irene for life, remainder to Daniel if he survives
Irene.
3. Vested Remainders Subject to Partial Divestment - A future interest is subject to
partial divestment when the interest is given to a group of people that may increase
in size between the time the gift is made and the time it becomes possessory.
a. E.g. To Susan for life, remainder to Susans children. (Susan is still alive)
b. Long v. Long (Page 366) - Henry Long conveyed property to his three
children in a fee tail. To X, and the children of his body begotten, and their
heirs and assigns forever. Emma and Edward had children so they locked in
a fee simple absolute. Jessie never had children. Jessie quitclaimed his
interest to Rosella Long and Rosella long left it in her will to Marie and John
Brown who then quitclaimed it to Howard Long and his wife Esther.
i. Issue: What is the nature of the interest remaining in a grantor, Henry
Long, after the creation by deed of a fee tail estate which was conveyed to
his son Jessie Long?
ii. Holding: Reversion. Each child get s 1/3 share of the reversion. Jessie
effectively conveyed his reversion to Rosella Long.
(i) *Look at the family tree on page 367
G. The Trust
1. A devise that allows you to split the title of the property into two parallel and
simultaneous titles - the legal title and the equitable title
a. Legal title - given to the trustee who is charged with the responsibility of
holding and managing the property for the sole benefit of the beneficiaries
b. Equitable title - the beneficiaries hold this.
2. Dynastic trusts - trusts wealthy people set up for their descendants or other
relatives that last as long as the law allows.
a. In some states, it is possible to set up perpetual dynastic trusts and in others
90-year trusts due to recent changes in the Rule.
H. The Rule Against Perpetuities and Other Doctrines that Destroy Future Interests
1. Why? Marketability and development of land
2. Rule in Shelleys Case - converted a remainder in the heirs of the grantee into a
vested remainder in the grantee.
a. To A for life, remainder to As heirs. Gave A a fee simple absolute.
3. Doctrine of Worthier Title - converted a remainder in the grantors heirs to a
reversion in the grantor.
a. From O To A for life, remainder to Os heirs. Life estate in A and a
reversion in O.
4. Destructibility of Contingent Remainders - destroyed remainders that were still
subject to a condition precedent when the prior estate terminated.
a. E.g. O to A for life, remainder to As children who reach 21. A dies when his
children are 17 and 18. The remainders are destroyed and O owned the
property in f.s.a.
5. The Rule Against Perpetuities
a. Does not apply to future interests held in the grantor - possibility of reverter,
rights of entry, or reversions.
b. Does not apply to future interests held by the government or charitable

organizations.
Only applies to executory interests, contingent remainders, and vested
subject to open.
d. No interest is good unless it must vest, if at all, not later than 21 years after
some life in being at the creation of the interest.
i. Need to determine if the interest will VEST or FAIL within the period.
e. Interests that violate the Rule are destroyed.
f. RAP Reform
i. Wait and See Test - wait and see what happens before deciding whether an
interest in void.
ii. USRAP - incorporates elements of wait and see and an irrebtuuable
presumption that a widow is a life in being. Provides an alternative 90year vesting period.
6. In Re Estate of Anderson (Page 380) - Conveyance: To the Trustee for the
education of the descendants of FA Anderson, Sr. for 25 years from the date of
admission of this will for probate, then to Howard Davis and if he is not alive, to
the heirs of his body.
a. Issue: Does this violate the RAP and if so should the future interests be
destroyed?
b. Holding: Yes it does violate the Rule but the court validates the trust anyway.
c. Reasoning: Principle that a persons will should be enforced so as to avoid
clearly unintended consequences. Court uses the wait and see test to save the
trust.
V. Concurrent Estates and Marital Property
A. Concurrent ownership: Tenancy in Common and Joint Tenancy
1. While you own property with someone else, you own undivided shares of property
each co-owner has a right to possess the whole
2. if you want to partition land youll have to reach agreement w/ co-owner or go to
court and get the court to partition land
3. If land transferred To A and B then presumption is that A and B will hold land as
tenants in common
B. Different Types of Co-Ownership
1. Tenancy in Common
a. interest in property is divisible either party may transfer their interest either
at death or inter vivos
b. If A and B own property, A can transfer interest to C either in As will or
during As lifetime
i. B and C will then hold f.s.a. in property as tenants in common
2. Joint Tenancy (with right of survivorship)
a. A and B hold property and B dies, As interest evaporates and B holds
property in f.s.a.
b. Property is not transferable either at death or inter vivos, but surviving party
has right of survivorship
c. If A transfers interest to C during life, A has severed joint tenancy and B and
C have tenancy in common
i. Can destroy other partys right of survivorship by transferring your interest
in blackacre
d. If A, B, and C have joint tenancy and A transfers property to D, then D is
tenant in common w/ B and C, but B and C still have joint tenancy
3. Tenancy by the entirety: limited to married couples
a. Property not transferable either at death or inter vivos
c.

b. Greatest form of protection against cotenants acts


c. only way to defeat tenancy by entirety is by divorce
d. strictly limited (in CA includes domestic partners)
4. Community Property (in CA)
a. if youre married and take property in any way, its assumed to be community
property
b. transferable at death, but not during life
C. Problems with Sharing Possession
D. Martin v. Martin (Page 557) D and P own property as tenants in common. Cotenant w/
7/8 interest developed mobile home park on property and collected rent from occupants.
Cotenant w/ 1/8 interest occupied of property and not paying rent.
1. Issue: When can one cotenant demand rent from another content?
a. Clearest case - If property rented out to third party, rent would be shared
between cotenants
b. In this case, party that owns 7/8 interest sought rent from party occupying part
of property
2. Occupying tenant also demanded accounting of rent proceeds from mobile home
park, which was ok.
3. What about expenses (security, utilities, taxes, etc.)?
a. Matter of equity so courts use their discretion
b. could say that occupying cotenants pay expenses of maintaining property
other than carrying costs that are clearly connected w/ protecting your
investment (mortgage, taxes, etc. these costs would be shared between
cotenants in proportion to their percentage of ownership)
c. costs such as utilities, upkeep, etc. should be paid by occupying cotenant to
eliminate conflicts over what type of improvements are made
4. Argument that non-occupying cotenant has been excluded:
a. Excluded from party of property where occupying tenant lives, but not from
entire property
b. Majority approach is that excluding from any part of property constitutes
exclusion
5. Ouster:
a. If ouster, non-occupying tenant entitled to rent
b. Must be absolutely clear that there was ouster; if any ambiguity then person
wont know theyve been ousted and clock starts running and they could lose
their interest to other tenant by AP
c. Safest thing to do if youve been ousted is to demand rent
6. Holding: Occupying co-tenants didnt owe rent to non-occupying co-tenants b/c
partial occupation of property doesnt constitute ouster
E. Yakavonis v. Tilton (Page 559) P and D ended relationship. Each retained interest in
properties A and B. B used as rental property and parties shared income. D lived in A.
1. Issue: Does occupying cotenant owe rent to non-occupying cotenant, and what
constitutes ouster?
2. P claims that carrying costs owed for maintaining property that D occupies should
be off-set by fair market rental value of property to be paid by D
3. Each cotenant has right to possess entire property, so only if non-occupying tenant
is not allowed on property (ouster) does that tenant have right to rent
4. Ouster:
a. this court says ouster affects possesory, but not ownership interest
b. so non-occupying tenant still has to pay carrying costs
5. How should courts evaluate costs for improvements to property?

a.
b.

Possible improvements could actually decrease value (e.g. purple house)


Most jurisdictions say both cotenants should share benefit of increased value
to property b/c of improvements, but if improvements decrease value then
cotenant that made improvements should bear the loss
6. Holding: Non-occupying cotenant is not entitle to rental value of property before
ousted, but is entitle to rent minus carrying costs after ouster
F. Delefino v. Vealencis (Page 563) P and D own property as tenants in common. D runs
waste management business on her part of property. Ps want partition by sale of
property to develop their portion.
1. Issue: Partition in Kind vs. Partition by Sale
a. Courts favor partition in kind
i. Concept of owelty: if we divide up property it rarely turns out that share
you get is exactly proportionate to your proportion of ownership, so
usually there is a financial reconciliation/buyout
ii. Uniqueness of blackacre and attachment to land
iii. Heightened preference for partition in kind when theres residence on land
b. Partition by Sale
i. party requesting partition has burden of justifying it
ii. Sometimes preferred b/c partition in kind is unwieldy given the attributes
of the property
2. Holding: Interests of both parties would be best served by partition in kind, which
is feasible.
G. Joint Tenancy with Right of Survivorship: Creation and Severance
1. Downing v. Downing (Page 570) Helen and son held property in joint tenancy.
D allowed third party to grow and harvest crops on property.
a. P, Helens daughter, claims farm was held as tenancy in common, not joint
tenancy, and that Helens interest passes to her as Helens heir
i. If joint tenancy, son would be sole owner b/c no right of survivorship
b. Arguments supporting joint tenancy
i. Expressed intent in language of conveyance (as joint tenants)
ii. Helen and son took out mortgage together
iii. Four unities: time, title, interest, possession
(i) All met except possession, b/c Helen had agreement to allow third
party to use property
c. Arguments for tenancy in common:
i. Conveyance didnt include words right of survivorship (necessary in
some jurisdictions)
ii. Helen allowed third party to use land for harvesting
d. Holding: Helen and son had joint tenancy
2. People v. Nogarr (Page 574) Calvert and Elaine were married and owned
property as joint tenants. After they separated (but never divorced) C took out
mortgage in property and transferred proceeds to his parents.
a. Issue: Did C sever joint tenancy by taking out mortgage w/o cotenant?
i. Court said taking out mortgage didnt sever joint tenancy
ii. Look at intent of parties when committing act rather than just the act itself
to see if they intended to sever
3. Smolen v. Smolen (Page 578) Roslyn and Martin married and own property.
Martin contracted brain disease. Parties divorced to protect assets. R moved M to
group home and had him declared incompetent. M established trust w/ nephew as
sole beneficiary. M dies.
a. R claims that Ms creation of trust violated divorce decree which stated that

real property shall remain in joint tenancy w/ right of survivorship


If court wanted to be clear that property would remain as joint tenancy forever
they couldve conveyed property: To R and M as joint life tenants, remainder
in f.s.a. to the survivor
i. b/c court didnt do this Jason had severance possibilities and creation in
trust was valid
c. Holding: Creation of trust severed joint tenancy. R and Ms nephew, as
beneficiary of trust, hold property as tenants in common
VI. The Recording System
A. Introduction: The common law rule was that a grantee who was prior in time prevailed
over one subsequent in time.
1. To discourage the grantor from selling his property more than once, statutes
regarding deed recordation were passed in all states in order to protect a grantee
from subsequent purchasers. .
2. Recording acts attempt to resolve the competing claims between first in time vs.
second in time purchasers.
3. Recordation is ONLY an issue when there is more than one grantee contesting title.
4. Recordation can only help the second in time purchaser.
5. If a person does not record, he does not come within the protection of the
recording acts and the common law rule of prior in time is applicable.
6. What constitutes recordation? You must record in a manner complying with the
applicable statute or judical decisions. If not, then you have not recorded. See
Patience v. Snyder.
7. As a court, when looking at these problems you must look at either fairness (who
is in your court room?) or chain of title (where did they get their land from?).
B. The Mechanics of Recording
1. The Grantor-Grantee Index: Indexed by the name of the grantor and grantee in
alphabetical order.
a. For a title search, look up the name of the person who sold you your land in
the grantee index book (because he was once a grantee) to find who was the
grantor in that transaction. You can trace backwards to the original grantor
this way or trace forwards to discover all grantees of your parcel of land.
2. The Tract System: Indexed by tract of land where property is located; look up all
of the transactions associated w/ particular plot of land.
C. Types of Recording Acts
1. Race Statutes Whoever records first wins. Notice is irrelevant.
a. must have paid valuable consideration AND recorded first
b. Ex: No conveyance or mortgage of an interest in land is valid against any
subsequent purchaser whose conveyance is first recorded.
2. Notice StatutesProtected if have no actual or constructive notice of prior claim
a. must have paid valuable consideration AND gotten no notice at the time of
the conveyance
b. protected whether he records at all
c. Ex: No conveyance or mortgage of an interest in land is valid against any
subsequent purchaser for value without notice thereof, unless it is recorded.
3. Race-notice Statutes must have paid valuable consideration AND no notice
AND recorded before prior deed was recorded
a. The subsequent purchaser must both be without notice AND win the race
record in order to win.
b.

b.

Ex: No conveyance or mortgage of an interest in land is valid against any


subsequent purchaser for value without notice thereof whose conveyance is
first recorded.
4. Period of Grace statutesGives the prior grantee a period of time in which to
record. Uncommon today.
5. For all statutes, dispute is resolved in favor of LAST good faith purchaser
D. Types of Notice
1. Record Noticenotice of instruments properly recorded within the chain of
title of the property, discoverable by a reasonable search of the records.
a. Ryczkowski v. Chelsea Title & Guaranty Co. (Page 751)JJ Cleary gives
utility easement to a power company across his land (they record). Later, he
receives the patent deed to his land from the government (and records). Now
the present owners want to know, are they bound by the easement to the
power company?
a. No! No competent title searcher should have the burden of having to
search record beyond point when grantor became a grantee (before
the land was owned).
b. Future land purchasers could not see the easement unless the power
company re-recorded after the patent to JJ Cleary went through.
c. Recording outside the chain of title creates a wild deed. You can
do this by recording too early, too late, or under a different name.
b. Morris v. Curtis (Page 752)Issue: Did C have constructive notice of H
D?
8/8/1872
Hall mortgage to D, no record
9/7/1872
Hall mortgage to Clark, notice of H D
1/31/1876
C records
9/8/1876
D records
10/4/1881
C T, no actual notice of H D
a. No! If C had done a competent search he could not have found
OA because records not organized chronologically by time of
conveyance, just by time of recordation.
b. C not obligated to search for conveyance recorded after HC was
recorded
c. Its too difficult to find people who are too late to record or who
record before they have title, and so we dont care about these people
because they could have prevented these problems from happening.
c. Buffalo Academy of the Sacred Heart v. Boehm Bros (p. 754) P offers D
property for sale (to pay off debt), and if property ends up being
unmarketable then P will pay D $60,000
d. Issue: O has made many conveyances, some with restrictions, some without.
Should downstream purchasers be considered to have constructive notice
about conveyances that are not in their own chain of title?
a. No! In the absence of actual notice, an owner of land is only bound
by restrictions if they appear in some deed of record in his direct
chain of title.
b. If grantor does not clearly put in everybodys chain of title
restrictions such that someone can find them, then they are not
enforceable restrictions. Courts differ on whether there is a duty of
inquiry notice in cases like this.

2. Inquiry NoticeForm of constructive notice. Under certain circumstances a


purchaser is required by law to make reasonable inquiries regarding land, and then
he is deemed to have had notice of whatever the inquiry would have revealed.
Burden of inquiry notice depends on the jurisdiction.
a. Sanborn v. McLean (Page 759)-- Grantor put restrictions on some deeds but
not others. Everyone acted like restrictions were on all deeds, protecting the
uniform plan that they understood to be created. D decides to build against
the restriction.
i. Would a reasonable person have asked and found out whether restrictions
existed?
ii. Holding: Yes! There was a duty of inquiry notice in this case, should have
read or asked about the contents of the other deeds out from the common
grantor.
E. When a Person is a Purchaser
1. Manufacturers and Traders Trust Co. v. First National Bank (Page 763)
DebtorA a mortgage, A pays valuable consideration. DebtorB the same
mortgage, says B can just owe him the same promise to pay that B has already
given to the debtor.
a. Issue: Is B a second in time subsequent purchaser of the property?
b. No more consideration is being paid and therefore not a second in time
purchaser for value, and the recording acts will not protect him!
c. However, if there is forbearance, then it will not be the same consideration as
before, and then B would have paid valuable consideration.
VII. Nuisance
A. Main problem: someone gets entitlement at someone else's expense, giving someone else
more rights
B. Different ways to conceptualize nuisance non-trespassory invasion
1. How invasive of Bs use is As use?
a. Consider how reasonable person would react
b. Invasive nuisance
c. Consider diminution of property value, but that alone is not enough
2. What is the social utility of each partys use?
a. Useful no nuisance
b. Appropriate remedies?
c. Consider suitability of use to locality and impracticability in
preventing/avoiding the invasion
3. First in time/Coming to the Nuisance
4. Intentional and Unreasonable vs. Unintentional but otherwise actionable
C. 4 Remedies for Nuisance
1. Allow the "nuisance (i.e. Clark)
2. Enjoin the "nuisance" stop nuisance
3. Allow the "nuisance," if the nuisance causer pays the complainer
4. Hold out potential for complainer so will say how much to pay/worth forced
transaction (i.e. Boomer)
5. Enjoin the nuisance, if the complainer pays the nuisance causer
a. If something so bad that enjoin it, why should you allow it? (Spur)
D. Public nuisance:
1. Private individual can claim only if can show that the nuisance is specially
injurious to them
2. The person does not have to own any affected land, but must show that the damage

E.
F.

G.

H.

I.

J.

to them is of a different kind than the damage to the public at large, not enough to
show that they suffer the same kind of harm as the general public but to a greater
extent
Private nuisance: Affects a single person or small number of people in enjoyment of
private rights not common to public
Clark v. Wambold (p. 841) - P sued to enjoin D from maintain pig yard on property
bordering Ps
1. matter of equity: both parties use rights are premised on their black letter property
rights
2. trial court found piggery sanitary and didnt materially interfere w/ Ps enjoyment
of land
3. piggery first in time and country only place D can run the piggery
4. P purchased property from D so knew about piggery
5. P only using the property as summer residence whereas D using to maintain
livelihood
6. What does it mean to each person to use property as he wants and to prevent other
from using property as he wants?
7. Holding: Piggery not a nuisance; encourages neighborliness
Mitchell v. Hines (p. 842) P sues to enjoin use of neighboring piggery. D feeding pigs
with garbage, spreading dead animals over field public health concern
1. Piggery use increased since P purchased property made odors horrible
2. Harms to P: flies increased risk of disease and diminished property value
3. Holding: Nuisance piggery enjoined
Boomer v. Atlantic Cement Co. (p. 851) - P sues to enjoin D's cement plant b/c of injury
to P from dirt, smoke, and vibration emanation from plant
1. injunction denied in trial court though a nuisance was found b/c damage to P was
small compared to the value of D's activity social utility argument
2. Holding: Court enjoins nuisance until D pays P permanent damages for present
and future economic loss
a. Incentivizes improving technology and puts burden on D
b. Eliminate possibility of future nuisance claim by buying P out
3. Alternative Solutions:
a. grant injunction but postpone effective date to give opportunity for technical
advances
b. Give bargaining power to downstream user to demand price
4. Dissent: like eminent domain: forcing transaction of private person A and B in
benefit of person B
When courts grant injunctions they presume:
1. low transaction costs (not if lots of people can sue the person)
2. rational entities
a. not necessarily many don't bargain b/c annoyed
b. hold out problems and have cement plant move out
3. more appropriate for parties to value than court
Spur Industries, Inc. v. Del E. Webb Development Co. (p. 858) - Appeal from judgment
permanently enjoining P from operating cattle farm near Ds development
1. "coming to the nuisance"
a. would have barred recovery if D only injured party, but individual
homeowners injured
b. court concerned w/ rights of public
2. D got windfall from buying cheap land, so why should it benefit when P was there

first?
a. Why cant D built buffer zone? Would give D more property than they own
b. Problem w/ nuisance law: giving one party entitlement to part of others
property
3. Holding: D gets injunction but has to pay the cost of relocating Spur
K. Prah v. Maretti (p. 867) - Owner of solar-heated residence sued to enjoin neighbors
proposed construction of residence that would interfere with P's access to unobstructed
sunlight across D's property.
1. By favoring solar power, court incentivizes its use
2. Sunlight only a matter of aesthetics?
3. Problem w/ Notice: Would reasonable purchaser see solar panels anywhere and ask
what they mean about neighbors/restrictions on property?
4. First in time vs. second in time
a. Ex: Unrestricted downhill lot builds up and now blocks someone's view in
uphill
5. Holding: unreasonable obstruction of access to sunlight might be private nuisance
claim, but court makes no determination
a. *Hinge point for most nuisance cases is what is most socially beneficial
use?
L. Mark v. State Department of Fish and Wildlife (p. 879) - P sues D for injunctive relief
and inverse condemnation for failure to control public nudity in adjacent wildlife area
1. public/private nuisance
a. failure by D to enforce own rules of buffer zones
b. court doesnt find public nuisance lack of statute saying nudity is a nuisance
c. nudity vs. nudity w/ intent to arouse
2. Could say that purchasers were hypersensitive b/c didn't bother anyone else - P
interfering user?
3. Injunction (only remedy available) municipal immunity from damages, but not
from injunctive relief
4. Inverse Condemnation? NO
a. property still has value since P living on it not being able to have visitors
over not enough to constitute taking
5. Holding: Private nuisance claim for injunctive relief remanded - nudity and sexual
activity impair Ps use and enjoyment of their property
VIII. Servitudes and Common Interest Communities
A. Running with the Land and Running with Interests in the Land
1. Servitudes are used to create rights (benefits) and obligations (burdens) that run
with the land burden or benefit automatically passes to subsequent owners.
2. Appurtenant: Benefits or burdens that run with the land
3. In gross: benefits or burdens that do not run with the land
a. does NOT = personal. May still be assigned to others
B. Types of Servitudes:
1. Easementscreates a right to enter and use land belonging to another and
obligates the landowner to refrain from interfering with this use
a. Profit a prendreadds to an easement the right to remove a natural resource
or hunt
2. CovenantsCreates a promise to do or not do something on your own land.
Personally liable for money damages.
a. Equitable servitudea covenant that is enforceable by injunction/equity,
historically shares characteristics with an easement
3. Easements and ProfitsCreate non-possessory rights/use rights.

a. Servient/burdened owner.
b. Dominant/benefited owner.
C. Creation of Easements and Profits
D. Creation by express grant. Statute of Frauds - agreements regarding easements need to
be in writing, signed by the grantor (unless by estoppel, implication or prescription).
1. Look at language of deed, together with circumstances, to determine intent of
parties.
2. Brown v. The Penn Central Corporation (Page 894) - Ps gave use of land to RR
for depot and railroad purposes, Ps wanted land back after RR stopped using it.
a. Issue: Did this deed create an easement or a fee?
b. Grant for a limited purpose, or a space without clearly marked boundaries
creates an easement.
c. Easement extinguished when RR stopped using it because of conditional
language (so long as). Ps could quiet title in land due to RRs
abandonment. Abandonment is a termination of an easement.
d. Better to convey easements than estates to RRs bc small strips of land far
apart from each other are not very marketable/practical.
e. Holding: This was an easement and not an fsa.
3. Stratis v. Doyle (Page 897) Did Ds create license or easement when conveyed to
Ps predecessor in interest for use as a driveway?
a. Warranty deed created an interest in land (grant, words of inheritance, etc).
b. Ds argued easement was subject to condition subsequent, requiring P to
maintain driveway. Not conditional b/c no right of re-entry language.
i. No possibility of forfeiture, but can get damages/contractual remedies for
its poor condition
ii. License=permission to go on land. Revocable at the will of the licensor.
c. Holding: Easement!
4. Cooper v. Boise Church of Christ (Page 899) - Issue: Did this deed conveying
right to church to erect sign on Ps property create an easement or a license?
a. Nominal consideration, no words of succession, rights given for limited
purposesuggests license (no one in their right mind would sell permanent
easement for 1$ ).
i. a failed attempt to create an easement may still result in a license!
b. Holding: No easement, a license.
E. Ways to create easement w/o writing
1. Creation by Estoppel: Elements to an estoppel claim:
a. A has made representation that he doesnt intend to stand on his black letter
property rights to require agreement in writing
b. Reasonable for B to rely on representation made by A and to assume that A
would stand on it
c. Reasonable for B to change behavior based on reliance of representation made
by A
d. B has changed his behavior b/c of reliance on As representation
e. B is substantially harmed
2. Mund v. English (Page 903) - Ds are parents and Ps are their kids. Both paid for
well to be built on Ds property. D now says he can revoke this right to use the
well.
a. Ds said Ps could use well, both paid to build and maintain the well, it was
reasonable for Ps to believe this was a legitimate offer
b. if Ds allowed to change their minds, Ps would be injusticed.

c.

Holding: Easement created by estoppel! Ps have interest in the well and


interest in maintaining/sharing the cost of the system.
3. Implied on the Basis of Prior Use: lack of writing can be excused if(some
jurisdictions require writing no matter what)
a. Properties were previously owned by common owner
b. Prior owner used one part of property for benefit of other part of property
c. Use was apparent to future purchasers and was necessary for utility of
dominant property
d. Amount of consideration was paid that reflected burden/benefit situation
4. Van Sandt v. Royster (Page 906) - Action brought to enjoin Ds from
using/maintaining underground sewer drains through and across Ps land.
a. All above requirements met
b. Jurisdictions differ as to whether they require strict necessity or only
reasonable necessity in order for grantor to benefit from easement if theres no
writing.
c. Difference between this easement and easement by necessity: Necessity a
FACTOR here, but not required. If it no longer becomes necessary for
sewage to run through these properties, the easement implied from prior use
does not end according to common law. Runs indefinitely!
d. Holding: Easement implied on the basis of prior use of lateral sewer drain.
5. Implied on the Basis of Necessity -- only with landlocked land.
6. Morrell v. Rice (Page 911) - Grantor conveyed land to D and P at same time. P
wants easement by necessity over Ds land b/c landlocked, surrounded by water
and swampland.
a. Easement by necessity must originate where grantor gives land to these
people simultaneously. As Grantor, if you create landlocked land your land
must be burdened by easement if grantor still owns any land.
b. If land becomes landlocked later, then tough luck, must bargain for an
easement from someone.
c. Govt will sometimes force easement by condemnation (its for a public good
because landlocked land is unmarketable), but questionable.
d. Holding: Easement created by necessity.
7. Creation by Prescription
a. prescriptive easements generally confined most closely to original use in
terms of scope. Need open and notorious use, under a claim of right (no
permission), continuous, and uninterrupted (by owners entry upon land, suit,
etc).
8. Paxson v. Glovitz (Page 917) - Oral easement created awhile ago for Ps
successors in interest to have an easement across Ds land. D finally builds a fence
blocking easement.
a. D says the use was not hostile b/c was permissive, but the court doesnt buy
this distinction. If an imperfect agreement is made (no writing) can cure this
problem by prescription.
b. Holding: P earned this easement by prescription.
F. Creation by Dedication, Condemnation, and Other Forced Sales
1. Goulding v. Cook (Page 921) - P sought to enjoin Ds to remove septic tank
trespass from Ps property.
a. Only possible location for a septic tank is on some of Ps property.
2. Can only force easements if absolute necessity (no landlocked land, and no
common grantor).

3. Holding: Septic tank is a permanent physical occupation on Ps property. No


easement. We cant force neighborliness. Try to work it out amongst yourselves.
Ps must remove septic system and pay damages.
4. What can Ds do? What bargaining power do they have?
G. Creating Easements for Third Parties
1. Estate of Thompson v. Wade (Page 925) - Noble conveyed two attached plots of
land to D and P, one along a road, one along a river. P claims easement over Ds
property.
2. Afterwards, he reserved easements over roadside property to other property
3. Holding : No easement! These easements are not valid because at the time Noble
created them, he no longer owned the land, was a stranger to the deed. Can only
reserve easements in the grantor.
4. If he had created the easement before selling either lots, that would be okay.
MUST be a two step process (create easement and then convey). Makes it easier
for second in time purchasers of land to find burdens on the land/whether the land
is burdened.
H. Scope of Easements and Profits
1. Location, Relocation, and Use of Easements: Default rule is that you may not
change the location of an easement.
2. Restatement (third) says the owner of the servient estate is entitled to make
reasonable changes in the location or dimensions of the easement, but only if
changes do not: lessen the utility of the easement, increase the burdens on the
owner of the easement, or frustrate the purpose for which the easement was
created.
3. Davis v. Bruk (Page 929) - D (Bruk) wants to relocate Ps right of way through
Ds property. P wants to pave the existing easement because gravel is hard to
maintain, and also this may make the right of way less dangerous to D.
a. No undue burden on P to change the right of way, new right of way more
convenient, D willing to pay for costs to move it.
b. Relocation may affect values of the servient and dominant estates
c. Holding: Cannot relocate right of way! D had knowledge of easement when
bought property. Need mutual consent of both people to relocate.
i. Ps cannot pave right of way. You got what you paid for. Cannot disturb
the land/change the easement. Paving the road might lead to an increase of
traffic and speed of traffic which would probably lead to change of scope
problems.
d. No need for court to give either party more bargaining/holdout power. There
is incentive for each person to bargain. Strength of default rule still salient!
4. Lewis v. Young (Page 932) - Issue: Can landowner (servient tenement) unilaterally
relocate an easement holders right of way over the burdened premises? D wants
to move Ps driveway across Ds property.
a. The intent of the original grantor of the easement did not seem concerned
about where the exact location of the easement should be.
b. Holding: Landowner can relocate right of way if
i. easement holders right of access is not impaired,
ii. D bears relocation expense,
iii. the utility of the right of way is not lessened, and
iv. it doesnt change the intent of creating the right of way.
c. Must negotiate with easement holder for destination change. If landowner has
the power to move an easement they will be more likely to improve the land,
raise marketability.

d.

Continued use of the easement (its been around for over 37 years!) cannot
prevent the landowner from moving it.
e. In these cases, consider who is moving the easement: the servient estate or the
dominant estate. Makes a difference in whether you think it is reasonable to
do!
I. Use of Easement for Non-Dominant Land
1. Brown v. Voss (Page 939) - Lots A, B, and C next to each other. Lot B has an
easement across lot As land. Lots B and C merge together. A wants to enjoin B
from using the easement to get to lot C.
a. Holding: Easement meant for B to go across As land. Thats what it was
created for, and thats how it should stay. No easement for C across A. Yes a
misuse of the easement.
b. However, court does not grant an injunction (Go to court for trespass and get
damages?).
J. Succession, Exclusivity, Assignability, and Divisibility
1. City of Pasadena v. California-Michigan Land and Water Co. (Page 944) - Two
competing water service companies get easements for water mains below land. P
got them first, says our easements were exclusive, must remove Ds water mains!
a. Holding: Easement didnt convey exclusive right, but right to use land for
reasonable and necessary use of easement. However, if it becomes necessary
for P to use the entire easement, then they have a prior right to the easement
than their competitor does.
2. Fairbrother v. Adams (Page 947) - Issues: Did the warranty deed convey
exclusive hunting and fishing rights? What is the scope of the rights? Are they
alienable and assignable?
a. Holding: Rights exclusive to the lot holders because in conveyance said right
to the h/f rights. It is associated with land, part of deed, and so an interest
in land.
b. Restatement: look at intent of the parties as to whether they wanted the burden
to run if the benefit was not associated with land.
K. Modification and Termination of Easements and Profits
1. Central Oregon Fabricators, Inc. v. Hudspeth (Page 950) - Profit given to D to
hunt and fish on land on which Ps hunting business is located. Ds never used
profit rights, so then planned to sell them to willing buyers. P wishes to quiet title
to land.
a. No intent that rights will be transferable if it would change the scope of the
profit. No intent to be given to paying guests, just personal guests
b. No termination of profit because no abandonment and no AP.
c. Holding: P cannot quiet title. Profit is not terminated. However, Ds cannot
sell profit rights to others.
d. AbandonmentDs never used their profit. However, court says there must
be a verbal expression of intent to abandon, and thus this is not abandonment.
e. AP/prescription claimeven less valid than abandonment. No hostile
possession. When P put up fences, kicked out trespassers, it was in the Ds
best interest.
I.

Covenants
A. Covenant: promise to do/not to do something
1. affirmative burden: promise to do something
2. negative burden: promise not to do something

B. Negative Easements in English Law obligate a landowner not to do something on his


own land that would interfere with the ordinary use and enjoyment of a neighbors land.
(p. 959) - Now treated the same as covenants
1. Easements of view
2. Conservation easements
3. Solar easements
C. Real covenant: covenant that runs w/ the land
D. Equitable servitude: doesnt run w/ land; can be implied
E. Remedies
1. breach of real covenant punishable by monetary damages
2. breach of equitable servitude enforceable in equity, by injunction or decree
F. Traditional Requirements for creating running covenants
1. all requirements must be met for real covenants b/c landowner subject to personal
liability/monetary damages if breach, but not for equitable servitudes
Requirement
Real Covenant
Equitable Servitude
Intent
Required
Required
Horizontal Privity
Required
Not required
Existence of General Plan
Not required
Not required
(that has been recorded or
come about by some other
way)
Writing
Required
Required, dont
necessarily need b/c
can be implied
servitudes or
servitudes by estoppel
Benefits in Gross
Probably OK, but
Not generally allowed
- difficulty in finding party
originally disfavored
or favored, but
w/ benefit made courts
allowed in some
disfavor benefits in gross
circumstances
- ok if theres a great benefit
Affirmative burdens
O
Ok, but some
- originally seen as not
exceptions
touching and concerning
land except in very specific
cases
- more burdensome to
require someone to do
something, than to not allow
someone to do something
Touch and concern
Required
Required
- affects owners legal
property rights, not rights as
individual
- e.g. increases/decreases
property values
Vertical Privity
- If you want burden to
No strict vertical
run to successors in
privity, b/c no money
interest, side w/ burden damage exposure
has to have complete

Requirement

Real Covenant
vertical privity

Equitable Servitude

Notice, also recording

Required

Required

G. Restatement (Third) Requirements for creating running covenants


1. no distinction between real covenants and equitable servitudes
a. all remedies available for either category
2. Only requirements:
a. intent
b. writing
c. vertical privity (only required for affirmative burdens
d. Notice (as per Recording Acts)
3. Not required:
a. horizontal privity
b. existence of general plan
c. touch and concern
4. Benefits in gross and Affirmative Burdens are OK
H. Covenants and Negative Easements in America
1. Runyon v. Paley (p. 962) Gaskins P Ruyons; Gaskins Brughs, subject to
certain restrictions allowing only residential use of the property.
Property retained by Gaskins P Williams.
Brughs D Paley, and began constructing condominium units.
a. Issue: Are the Ps entitled to enforce the restrictive covenants?
b. Four requirements to be a real covenant that runs with the land:
i. Touch and concern must affect the covenanting parties ownership
interests in the land; does not need to have a physical effect on the land,
may only have economic impact on the parties ownership right i.e.
enhancing the value of the dominant estate and decreasing the value of the
servient estate.
(i) must touch and concern the burdened and benefited land
(ii)Burdened estate: use restriction restricts Ds use and enjoyment of the
property and thus affects the value. Benefited estate: right to restrict
the use of Ds property affects Ps ownership interests in the property
owned by them.
1. Horizontal privity - privity of estate between the covenantor and
the convenantee at the time the covenant was created. The orginal
covenanting parties must create covenant in connection with the
conveyance of an estate in land from one of the parties to the other.
Grantor-grantee, servient owner-easement holder, and landlordtenant.
a. Yes covenants created in connection with the transfer of an
estate in fee simple.
2. Vertical privity The relationship between the original covenanting
parties and their successors in interest. Requires a showing of
succession in interest between the original covenanting parties and
the current owners of the dominant and servient estates.
a. D Paley and P Williams have succeeded to the estates once held
by the convenantor and covenantee Vertical privity is met.

b. P Runyons have not succeeded in any interest in land held by


Gaskins at the time the covenant was created no vertical
privity.
3. Intent - The intention of the parties that the burden and benefit run
at the time the deed containing the restriction was delivered. Look to
the language used, the nature of the restriction, the situation of the
parties, and the circumstances surrounding their transaction.
a. Did intend for it to run.
c. Can Ps Runyon enforce the covenant as an equitable servitude?
i. Touch and Concern Same analysis as above. Yes.
ii. Intent - If the party seeking enforcement was not an original party to the
covenant, he must show that the covenanting parties intended that he be
able to enforce the restriction Not sufficient evidence of this.
iii. Notice: restrictive covenant not enforceable, either in law or in equity,
against a subsequent purchaser of property burdened by the covenant
unless notice of the covenant is contained in an instrument in his chain of
title.
(i) P Williams has established that the public records provided sufficient
notice to Ds to enable her to enforce the restrictive covenants against
them.
(ii)Runyons records do not in any way suggest any right of enforcement
in favor of the Runyons.
d. Holding: P Williams is entitled to seek enforcement of the restrictive
covenants against Ds. Ps Runyons are not.
2. Sonoma Development, Inc. v. Miller (p. 976) Schaers owned two adjacent lots,
lot 38 and 39. Schaers sold lot 39 to Millers w/ restriction that nothing be built
w/in 3 ft. of wall between properties. Restriction recorded, and Schaers then sold
lot 39 to Sonoma.
a. Issue: Did horizontal privity exist between the original covenanting parties
(the Schaers and the Millers)?
i. Only the Schaers named as a party in the Declaration of Restriction.
ii. Declaration executed in conjunction w/ deed transferring property
b. Holding: Yes Court accepts substance over form. Covenant was part of a
transaction that included the transfer of an interest in the land.
3. Creation of Running Covenants 1) written instrument, 2) estoppel. 3)
implication, 4) prescription
a. Express Creation by Written Instrument
i. Created by either deed or declaration
ii. Identify the covenantor and convenantee
iii. Identify the property burdened by the covenant
iv. Identify the property benefited by the covenant if the benefit is appurtenant
v. Identity the parties intended to have enforcement rights if the benefit is in
gross
vi. State if the burdens, or benefits or both are intended to run with the land.
(i) All this information is ideal but not always necessary
b. Creation by Estoppel
i. Shamlimar Association v. D.O.C. Enterprises, Ltd. (p. 986) P retained
golf course lot and sold residential lots in development w/ restrictions as to
not interfere w/ golf course. No restriction recorded as to golf course lot.
D purchaser claims land is unrestricted.

c.
d.

e.

(i) Issue: Did D acquire an unrestricted fee? No


Can restrictions on the use of land arise other than by deed so as to bind
a purchaser w/ notice? Yes
1. no writing (no restriction in deed to D), but reliance of homeowners
on restriction, and compliance w/ restriction
2. D had actual and inquiry notice of restriction
3. D paid price that reflected restriction
4. Part performance: one party (homeowners) performed its part of the
bargain
Estoppel: action taken by one party (homeowners) which
demonstrates reliance on covenant
5. Parol evidence: evidence regarding deed which is outside the writing
(ii)Holding: Restrictive covenant requiring use of land for golf course
enforceable against D.
Covenants Created by Implication
i. Most implied covenants created when a developer sold property in a new
development
Covenants Created by Prescription and Eminent Domain
i. Prescription If landowners believe that they are bound by covenants and
perform the covenants for the prescriptive period, the covenants may
become validated by prescription even if there was some defect in their
creation (unrecorded covenants or recorded out of the chain of title)
ii. Eminent domain - the govt can impose a restriction by acquiring
covenant benefits.
Validity of Servitudes Are there certain kinds of rights or obligations that
people should not be allowed to tie to land ownership or occupancy?
i. Traditional: Imposed numerous limits on servitudes
ii. Restatement (Third): Aims at servitudes that pose a demonstrable risk of
harm to the public good
(i) Arbitrary, spiteful, or capricious
(ii)Unreasonably burdens a fundamental constitutional right
(iii)
Imposes an unreasonable restraint on alienation
(iv)
Imposes an unreasonable restraint on trade
(v) Unconscionable
iii. Open-Ended Covenants to Pay Money
(i) Neponsit Porperty Owners Assn, Inc. v. Emigrant Industrial
Savings Bank (p. 996) Lot owners are required to pay $4 a year to
the property owners association for maintenance of common areas such
as roads, paths, parks, etc. Bank refuses to pay.
1. Issue 1: Does this covenant to pay touch and concern land?
2. Holding: Yes Traditionally affirmative covenants to pay are not
allowed. Court recognizes some exceptions to this rule based on the
fact that covenant creates benefit to homeowners.
3. Issue 2: Is the vertical privity requirement met?
4. Holding: Yes - Not in the traditional sense. NPOA has not
succeeded to the ownership of any property of the grantor (Neponsit
Realty Co.). But has been formed as a convenient instrument by
which the property owners (who do hold property) may advance
their common interests.

(ii)Eagle Enterprises, Inc. v. Gross (p. 1001) - Covenant to pay $35 a


year for the seasonal supply of water. Gross build his own well so does
not want to pay for the water supply.
1. Issue: Is the promise of the original grantees to accept and make
payments for a seasonal water supply from the well of their grantors
enforceable against subsequent grantees?
2. Holding: No
3. Reasoning: Doesnt touch and concern the land does not
substantially affect ownership interests of landowners in the
subdivision.
a. Also, it is an affirmative covenant that creates a burden into
perpetuity.
b. Important point: Was this really the correct decision? What if
the parties were reversed?
iv. Restatement (Third) The Modification and Termination of Certain
Affirmative Covenants
(i) Terminates after a reasonable amount of time if the covenant does not
specify
(ii)Modified or terminated if obligation becomes excessive in relation to
the cost of providing the services/facilities or to the value received by
the burdened estate.
v. Covenants Not to Sue
(i) 1515-1519 Lakeview Boulevard Condominium Assn v. Apartment
Sales Corp. (p. 1005) - Homeowners of condos that were condemned
due to the soil sue the city. They argue that the city should not have
allowed to condos to be built due to the latent risk of soil movement.
Covenant in place that exculpates the city from liability for damages
caused by soil movement.
1. Issue 1: Does the covenant violate the abolition of sovereign
immunity?
2. Holding: No b/c the covenant was tailored to the specific risk;
although blanket grants of immunity are not allowed.
3. Issue 2: Does this exculpatory covenant recorded in the deed run
with the land? Does it touch and concern land?
4. Holding: Yes the covenant burdens the use of the land since it
limits rights usually associated with ownership.
vi. Restraints on Alienation
(i) Kerley v. Nu-West, Inc. (p. 1009) J & Resorco 15 acres
Kerley
Kerley $40,000 up front + promise to develop + promise to pay 10%
of each sale price Resorco
1. Issue: Does this constitute an unreasonable restraint on alienation?
2. Holding: No it is ok if it is reasonable under the circumstances.
Purpose to develop and sell the land. Kerley got the land for a
cheaper price creative financing. Duration the law implies that
it will be carried out within a reasonable amount of time.
(ii)Indirect restraints on alienation (Kerley case) Restatement (Third):
Controls the long term effects through its modification and termination
provisions.

(iii)
Direct restraints on alienation Restatment (Third): Invalid if
the restraint is unreasonable weigh the utility of the restraint against
the injurious consequences of enforcing the restraint.
a. Design Controls common feature of modern real estate developments.
Require submission of plans and approval by the developer or an
architectural control committee before construction begins. Usually valid power to disapprove plans must be exercised reasonably.
i. Rhue v. Cheyenne Homes, Inc. (p. 1028) Ps prevented from moving 30year old Spanish style house into new subdivision with only ranch style or
split level homes. Covenant specifying that it must be approved by the
architectural control committee.
Issue: Is this covenant enforceable given that it contains no specific
standards?
Holding: Yes Protects the value of nearby homes. Refusal to approve
plans must be reasonable, made in good faith and not arbitrary or capricious.
This was the case here.
*Some courts have held that design covenants are invalid unless they provide
sufficient guidelines to enable an owner to determine what is acceptable.
b. Protection Against Discrimination in Housing
i.

Protection Under the Federal Constitution


- 14th Amend. No state shall make or enforce any law which shall
abridge that privileges or immunities of citizens of the U.S.
a. Shelley v. Kraemer (p. 786) 30 property owners signed an
agreement restricting the use or occupancy of property in the area to
people of the Caucasian race.
Issue: Does the equal protection clause of the 14th Amend. allow judicial
enforcement by state courts of restrictive covenants based on race or
color?
Holding: No Cannot regulate agreements among private individuals
but the govt cannot enforce such covenants. Judicial action enforcing
these covenants amounts to govt action and is not allowed under the 14 th
Amend.
Practical effect: This case did not really do much. There is still the
intimidation factor.

ii.
-

Protection Under the Civil Rights Act of 1866 and the Federal
Fair Housing Act
Civil Rights Act of 1866 All citizens of the U.S. shall have the
same right, in every state and Territory, as is enjoyed by the white
citizens thereof to inherit, purchase, lease, sell, hold, and convey real
and personal property.
Federal Fair Housing Act (FFHA): more comprehensive than Civil
Rights Act b/c it covered discrimination on other grounds
i. Mrs. Murphy Exemption dont have to comply w/
FFHA if you are a small-scale entity selling/renting singlefamily house w/ an interest in the neighborhood

1. Cant force people to live next to people they dont


want to live next to
2. Under exemption, seller cant advertise sale
a. Jones v. Alfred H. Mayer Co. (p. 794) Mayer refused to sell Jones
a home for the sole reason that Jones is black.
Issue: Does the authority of Congress to enforce the 13th Amend.
include the power to eliminate all racial barriers to the acquisition of
real and personal property?
Holding: Yes Fair Housing Act some strict exemptions to this
(Mrs. Murphy). Need the Fair Housing Act b/c it is more
comprehensive than the Civil Rights Act which only deals with racial
discrimination.
c. Covenants Restricting Household Occupants and Personal Freedoms
Does servitude law impose limits on covenants that discriminate against
potential owners and occupants on grounds related to the composition of
their household and covenants that restrict personal freedoms?
i. Nahrstedt v. Lakeside Village Condominium Assn, Inc. (p. 1032) P
purchased a condo in assn and moved in with her 3 cats, claiming she was
unaware of restrictive covenant that disallowed pets. P wants to court to
declare the restriction unreasonable as applied to indoor cats that are not
allowed to run free in the projects common areas.
Issue: When can a condominium owner prevent enforcement of a use
restriction that has been recorded?
Holding: Court will uphold the decisions made by the governing board as
long as they represent good faith efforts to further the purposes of the
common interest development, are consistent with the developments
governing documents, and comply with public policy.
L. Presumption of validity when it comes to recorded use restrictions. Up
to the owner to prove that the restriction is unreasonable.
M. Enforcement must not be arbitrary or capricious and must be applied
uniformly.
N. Since reasonableness or unreasonableness is to be determined by
reference to the common interest development as a whole, a pet
restriction is not unreasonable as a matter of law.
ii. Solutions:
-Business judgment rule - presume that their behavior (HOA) was
reasonable unless you show one of three things 1) Conflict of interest; 2) Bad
faith; or 3) Arbitrary/capricious use of power. The burden is on the owner.
- Courts should review association decision making using the reasonableness
standard. Burden on the association to justify its action.
-Restatement 3rd Middle ground: Reasonable - owner bears the burden of
proving unreasonableness. Presumption of reasonability. Also, looks to
Where did the covenant first originate?
If in the master deed or original plan = strictly enforceable
If added later = subjective to reasonability issues (Grandfathered in)
5. Succession to Covenants: Vertical Privity and Restatement (Third)
- Succession to burdens: Traditional requirements distinguishes burdens that

run at law (strict vertical privity) and burdens in equity (relaxed vertical
privity).
- Successions to benefits: Traditional Relaxed vertical privity is required for
benefits to run at law and in equity.
- Restatement (Third) distinguishes between negative and affirmative covenants
such that vertical privity is only required for affirmative burdens.
6. Modification, Amendment, and Termination of Covenants
a. Changed Conditions
i. Rick v. West (p. 1044) Ps want a declaratory judgement to permit
the sale of 15 acres in the tract for a community hospital in spite of
covenants limiting the land to residential use. D refuses to consent to
release the covenants.
Issue: Can substantial changes in the neighborhood warrant declaring
the covenants unenforceable?
Holding: No Only changes since the covenants were filed are 2
commercial establishments not visible from the Ds property not
abutting Ps or Ds land.
ii. Restatement (Third) When a change has taken place since the creation
of a servitude that makes it impossible to accomplish the purpose for which
the servitude was created, a court may modify the servitude to permit the
purpose to be accomplish. If modification is not possible, then a court may
terminate the servitude and may award compensate of beneficiaries of that
servitude.
b. Amendment
i. Evergreen Highlands Association v. West (p. 1050) Original Evergreen
covenants provided that the majority of lot owners may modify the
covenants. 75% of the lot owners decided to add a new Article to the
covenants requiring all lot owners to be members of and pay assessments to
the Association. West bought his lot when membership and payment was
voluntary. He refuses to pay.
Issue 1: Is the modification clause of the covenant broad enough to allow for
the addition of a wholly new covenant by the requisite majority of property
owners?
Holding: Yes Follows the Zito line of cases look to the facts and the
consequences ok it this case b/c not unreasonable or burdensome.
Issue 2: Does the Association have an implied right to levy assessments
against lot owners in order to maintain common areas of the subdivision?
Holding: Yes follows the Restatement (Third) A common-interest
community has the power to raise the funds reasonably necessary to carry out
its functions by levying assessments against individually owned property in
the community even if not expressly granted by the declaration or by
statute. This is a common-interest community by implication so the
Association has the power to do this.
c. Termination
i. Westwood Homeowners Assn v. Lane County (p. 1058) - The county

acquired title to 15 lots in a planned community at a tax foreclosure sale. The


homeowners association made assessments against each of the lots for assn
fees that were incurred after the county acquired title to the 15 lots. The
county refused to pay the assessments and the homeowners association
brought a foreclosure action to enforce its liens.
Issue: Do liens and encumbrances as contained in the statute include
covenants?
Holding: No servitudes are excluded. It would have caused a lot of havoc
do not want covenant destroyed by foreclosure b/c there are still benefited
lots out there. Principle is the same as A.P. only APing against what the
owner had before you. Get the land with all of the restrictions.
ii. Traditional Termination Doctrines:
1. Buy a release from the benefited side
2. Expire as of a date in the document
3. Merger the burdened lot buys the benefited lot so that the benefit and
burden merge and covenant is released from land
4. Abandonment of the benefit more than just nonuse
5. Estoppel the person who held the benefit led the burdened side to
believe that it did not mean anything to them anymore and the they
reasonably relied on this and substantially changed position on this
reliance.
6. AP behave in opposition to covenant w/o opposition for statutory
period
7. Condemnation of the property govt not only pays the burdened
holder but also pays the benefited holder for the loss to them in violating
and taking the restriction.
8. Changed circumstances:
1.
Old def: Have a subdivision and changed
circumstances are happening wholly outside. Doctrine will not
apply.
2.
Softened approach: If one can make a
compelling case then court or legislature will lift the person out by:
1) forced transaction for a fixed price that had to be paid to the
benefit holders; 2) termination date by statute.
iii. New Forms:
1. Statutory modification of rights (cash-out)
2. Statutes that create expiration dates
3. Develops themselves retain power of modification and termination (e.g.
convince 75% of the people)
IX. Eminent Domain
A. Conventional Condemnation
- Right grounded in the constitution 5th Amend.
- Power of the state to force a private property owner to relinquish property to the State.
- 2 conditions must exist:
o Just compensation
o Public purpose

States allowed to do this too through the 14th Amend.


1. Public Purpose
a. Hawaii Housing Authority v. Midkiff (p. 1066) i. Issue: Does the public use clause of the 5th Amend. prohibit the State
of Hawaii from taking real property from lessors and transferring it
to lessees in order to reduce the concentration of fees simple in the
State, as promulgated by legislature?
ii. Public use? Public purpose to free up land and break oligopoly, even
though land wont be used by public
iii. Standard of review: legislative freedom to act
1. court only determines whether legislature acted rationally
2. Public use is to be determined by the legislature, not the
courts - Hands-off approach.
iv. Holding: ED allowed.
b. Poletown Neighborhood Council v. City of Detroit (p. 1071) City of
Detroit condemned Poletown to acquire the site for a GM site. The govt
claimed public purpose of economic redevelopment.
i. Issue: Is the acquisition for a public purpose?
ii. Holding: Yes the benefit to a private interest (GM) is incidental.
c. City of Oakland v. Oakland Raiders (p. 1075) Oakland wanted to keep
the Raiders in the City.
i. Issue: Is a team an acceptable property for condemnation under
state law?
ii. Holding: Yes Providing that the city can demonstrate a public
purpose.
1. Commerce Clause prevented the condemnation
2.

Justness of Just Compensation Rules


a.

The Procedure of Condemnation


- State statutes provide procedures that must be followed
- May be required to negotiate a purchase before formal proceedings
begin
- If negotiation fails, an appraisal of the property occurs Commission
of appraisers appointed by the court and the landowner may present
evidence of the propertys value

b.

Valuation
FMV when is this calculated?
Comparable properties Recent actual sales of substantially
similar properties
If govt pays before ED proceedings have gone through, seller
may get FMV+
Exceptional value? element that can be considered but no rights to this
Income flow value element that can be considered but no right to this
Replacement value cannot claim this; too difficult to determine and too

costly for govt to pay


Consequential damages the things associated with the govt taking the
land such as moving costs. Cannot claim this.
Fairest solution may be to ensure that FMV is calculated correctly

c. Claims of Inadequate Government Compensation for Condemnation


i. Mitchell v. United States (p. 1083) Govt condemning a tract of
land used by the Ps to grow and can corn of special grade quality. Ps
unable to reestablish themselves elsewhere in their former business.
- Issue: Should the Ps be compensated for the loss of this
business?
- Holding: Can consider this loss of business but dont have to
consider it as an element of compensation. If the business was
destroyed, the destruction was an unintended incident of the
taking of land so the Ps were not compensated for this loss.
ii. United States v. Fuller (p. 1084) Govt brought an eminent
domain proceeding against the Ps who operated a ranch on lands that
they owned in fee simple, land leased from the state, and land held
under the Taylor Grazing Act.
- Issue: Can a jury consider value accruing to the fee lands as a
result of their actual or potential use in combination with the
Taylor Grazing Act permit lands?
- On the open market the value of their fee lands was enhanced b/c
of the actual or potential use in conjunction with these permit
lands.
- Holding: Yes but such payment is not required. The 5th
Amend. does not require the govt to pay for the land based on
value derived from govt permit.
L. Inverse Condemnation A cause of action against govt to recover value of property
that has been taken in fact by govt, even though no formal exercise of eminent domain
has been attempted.
1. Requires a court to make a factual finding that the taking has occurred when the govt
insists that none was intended.
2. An action forcing the govt to purchase the Ps property initiated by the private
property owner.
1. Flyover
a. Griggs v. Allegheny County (p. 1088) D built airport bordering Ps
property and condemned land for that purpose. Planes pass closely over Ps
house.
- Issue: Has the county taken an air easement over Ps property for which
it must pay just compensation?
Doesnt need to be 100% taking for IC, can be taking of
easement
- Govt took strip of airspace from P
- No nuisance b/c of social utility of airport and state of the art technology
- Holding: Yes Ps house it no longer usable due to the constant noise

and vibrations from the aircraft.


2.

River Navigating and Flooding


a. Phelps v. Board of Supervisors of Muscatine (p. 1091) D constructed dam
which caused their lands to be flooded by impeding the free flow of the river.
There was flooding on Ps property prior to the construction.
Issue: Did the new causeway and bridge occasion a substantially greater degree
of flooding than the lands were subject to before?
Holding: Yes does not matter that the flooding is only intermittent when the
damage from it is permanent.

3.

Restricting Pollution in Rivers and Streams


c. United States v. 531.13 Acres of Land, Etc (p. 1092) U.S. condemned part
of Stevens property, where he operated a textile mill, to build the dam. To
protect the use of the reservoir the state required Stevens to upgrade the quality
of its waste water.
Issue: Does the U.S. owe Steven for the cost of the upgrading his disposal
system?
Holding: No Govt took nothing belonging to Stevens. He had no absolute
right to discharge industrial wastes into the Creek or River.

C. Condemnation of a Servitude
1. Condemnation of Easement and the Partial Takings Rule the taking of an
easement should be compensated. Compensation based on the difference between
the fair market value of the benefited property with and without the easement.
a. The Partial Takings Rule When the condemnation takes less than the full
property rights of the owner.
Fmv (before) Fmv (after) = just compensation
2. Condemnation of Restrictive Covenants and Affirmative Covenants to Pay
a. Arkansas State Highway Commission v. McNeill (p. 1095) - Govt took
property to build highway interchange bordering Ds property, which was
subject to restrictive covenant permitting only residential use of property. D
argues that benefit of neighboring residential property was taken and should
be compensated.
i. Issue: Does the fact that the proposed highway will violate the
restrictive covenant render the appellant liable for the decrease in
market value of the McNeills property?
ii. No harm to P in excess of harm suffered by general public
iii. Holding: No The same injury would have occurred in the absence
of the restrictive covenant. However, the court does recognize the
covenant as a property right.
b. Palm Beach County v. Cove Club Investors Ltd. (p. 1097) i. Issue: Does the right of a private country club to receive an income
stream from a monthly recreation fee assessed against the owner of a
residential mobile home lot constitute a property right compensable
upon inverse condemnation by the county for use of that lot in a
public road widening project?

ii. Holding: Yes - a restrictive covenant imposed a duty which ran with
the land taken which constituted a compensable interest.
c. Ways that courts have dealt with this area:
i. Real covenants are not property rights at all; they are contract rights.
ii. Whether they are property rights or not, govt is responsible only for
the rights attached to the land that is actually taken.
iii. May be property rights, but for purposes of ED law they are not
property rights.
iv. Some are property rights and some arent. Some types of covenants
will be compensated and some will not be (e.g. Palm Beach County
case)
v. All are property rights, but if the same injury would have occurred
without the covenant, then the govt does not have to compensate.
(e.g. Arkansas case)
X. Chapter 14 Land Use Regulation and Its Constitutional Limits
A. Regulation: When Does It Go Too Far?
1. The early cases
a. Pennsylvania Coal Co. v. Mahon (p. 1109) - A deed granted Ps the surface
rights to certain land but reserved to D the right to mine all coal under the house.
Plaintiffs argued that the Kohler Act (concerned with subsidence) extinguished
D's right to mine under Ps' surface land.
i. Issue: Can the legislatures police power be stretched this far?
ii. Holding: No - The Kohler Act was unconstitutional as a taking of
defendant's rights under a valid contract. In order to protect themselves,
plaintiffs should have contracted to acquire more than the surface rights.
The Kohler Act could not have been used to terminate the valid
contractual rights defendant received, nor could the Act could be used to
take defendant's contract rights without adequate compensation this
regulation constituted a taking.
iii. Dissent: This is nuisance regulation and if the govt is regulating a
nuisance then they do not have to pay.
b. Miller v. Schoene (p. 1113) Acting under the Cedar Rust Act of Virginia, D
ordered P tree owners to cut down a large number of ornamental red cedar trees
growing on their property as a means of preventing the communication of a rust
or plant disease with which they were infected to the apple orchards in the
vicinity.
i. Issue: Is this statute constitutional? Does it impose a taking that should
be compensated?
ii. Holding: No - The Court held that when forced to make the choice, the
state did not exceed its constitutional powers by deciding upon the
destruction of the cedar trees in order to save the apple orchards, which
in the judgment of the legislature was of greater value to the public.
This is a public nuisance = no compensation
2. The Modern Takings Cases
c. The 1987 Cases
i. Keystone Bituminous Coal Assn v. DeBemedoctis (p. 1142) Coal
Co. sought to enjoin officials of the Department of Environmental

Resources from enforcing the Subsidence Act alleging that it constituted


a taking of their private property without compensation.
1. Issue: Is the Subsidence Act constitutional?
2. Holding: Yes Difference from the Penn. Coal Co. case because
not taking 100% of something = no taking. Concept here is you
can mine out coal but you have to provide support. Public
purposes served by the Act. Also, the procedural posture of the
case is different.

You might also like